Find horizontal asymptotes of a radical function

Click For Summary
The discussion focuses on finding the horizontal asymptotes of the radical function f(x) = √(x² + 4x) - √(x² + x). Initial attempts involved factoring and conjugating the function, leading to a partial solution. A participant noted that by substituting large values for x, they determined the asymptote to be y = -1.5. Another contributor suggested rewriting the function to simplify the limit calculation as x approaches infinity. The conversation highlights the importance of proper algebraic manipulation in determining horizontal asymptotes.
LANS
Messages
24
Reaction score
0

Homework Statement


Find the horizontal asymptotes for the following equation:

Homework Equations


<br /> f(x) = \sqrt{x^2+4x}-\sqrt{x^2+x}<br />

The Attempt at a Solution


First I factored f(x):
<br /> f(x) = \sqrt{x}\sqrt{x+4}-\sqrt{x+1}<br />
Then I conjugated it:
<br /> f(x) = \frac{x(x+4-x+1)}{\sqrt{x}\sqrt{x+4}-\sqrt{x+1}}<br />
That's as far as I've been able to get. Any help would be appreciated.

edit: I "cheated" by plugging in big numbers and found the asymptote is y= -1.5
 
Last edited:
Physics news on Phys.org
LANS said:

Homework Statement


Find the horizontal asymptotes for the following equation:



Homework Equations


<br /> f(x) = \sqrt{x^2+4x}-\sqrt{x^2+x}<br />


The Attempt at a Solution


First I factored f(x):
<br /> f(x) = \sqrt{x}\sqrt{x+4}-\sqrt{x+1}<br />
Then I conjugated it:
<br /> f(x) = \frac{x[x+4-x+1]}{\sqrt{x}\sqrt{x+4}-\sqrt{x+1}}<br />
That's as far as I've been able to get. Any help would be appreciated.

edit: I "cheated" by plugging in big numbers and found the asymptote is y= -1.5
do you mean (plus sign on denominator & brackets)
f(x) = \frac{x[x+4-x+1]}{\sqrt{x}(\sqrt{x+4}+\sqrt{x+1})}

i would start with
f(x) = \frac{3x}{\sqrt{x^2+4x}+\sqrt{x^2+x}}

now try taking x outside the denominator and cancelling with numerator (or equivalently multiply both by 1/x)

then take the limit as x goes to +- infinity
 
Last edited:
I fixed the brackets, and I'll try that tomorrow (I'm going to bed now). Thanks.
 
Question: A clock's minute hand has length 4 and its hour hand has length 3. What is the distance between the tips at the moment when it is increasing most rapidly?(Putnam Exam Question) Answer: Making assumption that both the hands moves at constant angular velocities, the answer is ## \sqrt{7} .## But don't you think this assumption is somewhat doubtful and wrong?

Similar threads

  • · Replies 31 ·
2
Replies
31
Views
2K
  • · Replies 105 ·
4
Replies
105
Views
6K
  • · Replies 6 ·
Replies
6
Views
1K
  • · Replies 7 ·
Replies
7
Views
1K
Replies
4
Views
2K
  • · Replies 1 ·
Replies
1
Views
2K
  • · Replies 4 ·
Replies
4
Views
2K
Replies
2
Views
2K
  • · Replies 4 ·
Replies
4
Views
2K
  • · Replies 27 ·
Replies
27
Views
2K